Đến nội dung

fghost nội dung

Có 223 mục bởi fghost (Tìm giới hạn từ 26-05-2020)



Sắp theo                Sắp xếp  

#455388 Cho ánh xạ $f:X\to Y$, chứng minh $f$ toàn ánh khi v...

Đã gửi bởi fghost on 05-10-2013 - 20:25 trong Đại số đại cương

$f$ là song ánh từ $X$ vào $X$, như vậy $f$ có thể được xem như một phần tử của nhóm $S_n$. Và $f^{n!}$ phải là $1$ trong $S_n$, hay $f^{n!}=Id_X$




#455392 Chứng minh các đẳng thức sau :

Đã gửi bởi fghost on 05-10-2013 - 20:29 trong Đại số đại cương

Nếu $(x,y) \in (A\cap B)\times C$, thì $x \in A\cap B$ và $y \in C.$ Như vậy $x\in A$ và $x\in B$, nói cách khác, $(x,y)\in (A\times C)$ và $(x,y)\in B\times C$.

 

Nếu $(x,y)\in (A\times C) \cap (B\times C)$, thì $(x,y)\in A\times C$ và $B\times C$. Như vậy $x\in A$ và $x\in B$, hay $x\in A\cap B$. Và ta có $(x,y)\in (A\cap B) \times C$.

 

Bài 2 tương tự.




#456046 Cho ánh xạ $f:X\to Y$, chứng minh $f$ toàn ánh khi v...

Đã gửi bởi fghost on 08-10-2013 - 04:54 trong Đại số đại cương

Những bài này cơ bản, chỉ là diễn giải định nghĩa.

 

$'\Leftarrow'$. Nếu $y\in Y,$ thì $f(g(y))=y$. Như vậy $f$ toàn ánh.

 

$'\Rightarrow'$. Nếu $f$ toàn ánh, thì với mọi $y \in Y$ ta có $f(x_y)=y$ với $x_y$ nào đó trong $X$. Như vậy, định nghĩa $g: Y \rightarrow X$ với $g(y)=x_y.$ Rõ ràng $f\circ g= ID_Y$




#456545 Cho ánh xạ $f:X\to Y$, chứng minh $f$ toàn ánh khi v...

Đã gửi bởi fghost on 10-10-2013 - 08:47 trong Đại số đại cương

Bài giải của mình dễ hiểu hơn chứ nhỉ? :)




#460831 $S_3/S'_3 \cong Z_2$

Đã gửi bởi fghost on 30-10-2013 - 11:54 trong Đại số đại cương

1/ Với mọi $g \in G,$ ta cần chứng minh $a^{-1}G'a \subset G$. Ta có, với $c \in G',$ $a^{-1}ca=(a^{-1}cac^{-1})c=[a,c^{-1}]c\in G'c=G'$, điều phải chứng minh.

 

2/ $S_3$ có 6 phần tử là $1, (12),(23),(13),(123),(132)$. Viết ra những giao hoán tử, ta thấy đó là $\{(132), (123), 1\}$. Vì vậy $S'_3=<(123)>$.

 

3/ $|S_3/S'_3|=2$, mà nhóm có 2 phần tử chỉ có thể đồng dạng với $Z_2$. Điều phải chứng minh.




#461195 Xác định nhóm G

Đã gửi bởi fghost on 31-10-2013 - 22:42 trong Đại số đại cương

1.Xác định nhóm xích được sinh ra bởi $A$. Ta thấy, nhóm $<A>$ sẽ có 4 phần tử. Tương tự mỗi nhóm $<B>$ có 2 phần tử. Ta có quan hệ $BA=A^3B$ hay $BAB^{-1}=A^{-1}$. Như vậy, nhóm này là $D_4$ (nhóm diheral với 8 phần tử), $D_4=\{1, A, A^2, A^3, B, AB, A^2B, A^3B\}$. 

 

2. Các nhóm con của $D_4$ bao gồm $\{1\}, Z_2, Z_4, V_4, D_4$ tính đến đồng dạng.

 

Ghi hết ra thì như thế này.

$\{1\}$.

$Z_2 \cong <A^2> \cong <B> \cong <AB> \cong <A^2B> \cong <A^3B>.$

$Z_4 \cong <A>.$

$V_4 \cong \{1, B, A^2, A^2B\} \cong \{1, AB, A^2, A^3B\}$

$D_4$




#462189 không gian Mêtric

Đã gửi bởi fghost on 05-11-2013 - 05:50 trong Giải tích

Bài này trong topo mới phải.

 

Với mọi $b \in B$ cố định. Xét hàm $f: A \rightarrow R$ với $f(a)=d(a,b)$. Hàm $f$ liên tục, vì với mọi $\varepsilon >0$, $|d(a,b)-d(a',b)| \leq d(a,a') < \varepsilon.$ Vì $A$ compact, nên hàm $f$ sẽ đạt inf ở $x_0$ nào đó. Như vậy, $d(A,b)=d(x_0,b)$.

 

Lấy inf hay bên ta được $d(A,B)=\text{inf}\{d(A,b): b \in B\}=\text{inf}\{d(x_0,b): b\in B\}=d(x_0,B).$




#463121 $f(y)=y^{5}+y+1$

Đã gửi bởi fghost on 09-11-2013 - 20:53 trong Giải tích

Nếu $g$ là hàm ngược của $f$, thì $g(f(x))=x$ với mọi $x$. Lấy đạo hàm 2 bên, $g'(f(x))f'(x)=1$. Như vậy, $g'(1)=g'(f(0))=1/f'(0)=1$




#464337 Ví dụ về nhóm con pi-tựa chuẩn tắc

Đã gửi bởi fghost on 14-11-2013 - 20:05 trong Toán học hiện đại

Nhóm Diheral $D_4$ có 8 phần tử. $D_4=\{1, \sigma, \sigma^2, \sigma^3, \pi, \sigma \pi, \sigma^2 \pi, \sigma^3\pi \}$, $Z(D_4)=\{1, \sigma^2\}$. Nhóm $A=\{1, \pi\}$ $\pi$-tựa chuẩn tắc, nhưng không chuẩn tắc. 




#464430 Chia 15 hộp sữa cho 3 người

Đã gửi bởi fghost on 15-11-2013 - 03:22 trong Xác suất - Thống kê

Vì mỗi hộp sữa ngoại đều có xác suất được chia vào mỗi người như nhau và độc lập, như vậy mỗi khả năng chia là 1 nghiệm không âm nguyên của phương trình $a+b+c=3$, và mỗi nghiệm đều có khả năng xảy ra như nhau. Tổng số nghiệm là ${5 \choose 2}=10.$ Như vậy

 

(a) có 3 trường hợp, xác suất là $3/10$,

 

(b) có 1 trường hợp, xác suất là $1/10.$




#464754 Nghịch lý monty hall

Đã gửi bởi fghost on 17-11-2013 - 06:14 trong Nghịch lý

Xác suất có quà của A và B là như nhau và bằng $1/2$ vì mỗi hộp đều có khả năng có quà như nhau.

 

Nghĩ theo chiều hướng như thế này sẽ dễ hiểu hơn: bạn chọn 1 hộp trong 3 hộp rỗng. Mình sẽ để quà vào 1 trong 3 hộp đó ngẩu nhiên, thì xác suất hộp bạn chọn có quà là $1/3$. Vì MC sẽ loại bỏ 1 hộp không có quà, nên mình chỉ có 2 lựa chọn để đặt quà vào, và vì mình chọn ngẫu nhiên, nên xác suất có quà của bạn sẽ là $1/2$.




#465206 Nghịch lý monty hall

Đã gửi bởi fghost on 19-11-2013 - 08:30 trong Nghịch lý



Tôi lại có cách nhìn hơi khác. Giả sử ba hộp là A,B,C. Như vậy ban đầu xác suất có quà của mỗi hộp đều bằng 1/3.

Ban đầu ta chọn một hộp, giả sử là A, thế thì trong B và C luôn có ít nhất 1 hộp không có quà và người dẫn chương trình (MC) biết chắc điều này! MC mở hộp không có quà, giả sử đó là C. Bây giờ MC hỏi ta có đổi A với B hay không?

Thực ra nếu ta có đổi thì cũng chỉ là đổi cái xác suất 1/3 này lấy cái xác suất 1/3 kia mà thôi!

Nếu được quyền đổi, thì sau khi loại bỏ 1 hộp, thì chỉ còn 2 khả năng mà hộp quà ta chọn có quà (đổi hay không đổi: Nếu hộp A có quà, mà ta đổi, thì không được quà. Nếu hộp A có quà, mà ta không đổi thì ta được quà). Và vì việc đổi hay không đổi có xác suất xảy ra như nhau, nên xác suất hộp ta chọn có quà (sau khi quyết định đổi hay không đổi) là $1/2$. 

 

Còn nếu như không được quyền đổi, thì như bạn hxthanh đã đề cập, xác suất được quà là $1/3$ vì việc mở hộp không quà chỉ đơn giản là hành động kiểm tra mà thôi. (mình đã suy luận sai trong lần đầu)




#465359 Nghịch lý monty hall

Đã gửi bởi fghost on 19-11-2013 - 21:29 trong Nghịch lý



mình nghĩ ở đây mình không đề cập đến việc có đổi quà hay là không, bản chất nó là xác định xác suất CÓ QUÀ của từng hộp A và B sau khi MC mở hộp rỗng C kia thôi.

 

Mình thấy bài đầu của bạn có câu này

 

 

Hỏi người chơi có nên đổi hộp không.

 

Việc cho phép người chơi đổi quà làm thay đổi xác suất có quà (như giải thích bên dưới). Còn nếu người chơi không được phép đổi, hay bắt buộc phải đổi, thì xác suất sẽ khác đi.

 



mình có đính chính lại như sau:

-giả sử nếu ko đổi hộp thì xác suất trúng cũng chỉ là $\frac{1}{3}$ bời vì đơn giản mc chỉ bỏ đi 1 hộp ko có quà ko phải hộp mình chon nên hôp chọn được bảo toàn$\Rightarrow$ xác suất được bảo toàn

-còn về xác suất khi đổi thì vẫn khẳng định xác suất là $\frac{2}{3}$.Đây có thể là cách giải đễ hiểu hơn cách giải trước:

+tương tự ta có 3 hộp: k,k,c.

+theo đề bài thi khi mc bỏ đi 1 hộp k thì nếu trước đó ta chọn hộp k còn lại thì khi đổi ta chắc chắn được hộp c.vậy để tính xác suất khi đổi ta được hộp c thì phải tính  xác suất ta chọn được hộp k lúc đầu.

+xác suất chọn lúc hôp k khi mc chưa lấy hộp là $\frac{2}{3}$

+nên nếu mc lấy đi 1 hộp k rồi thì khi đổi ta chắc chắn được hộp c.

Vậy xác suất khi đổi hộp là $\frac{2}{3}$

 

Bạn giải thiếu rồi hay có lẽ đề không rõ và mình hiểu nhầm, người chơi có quyền đổi hay không đổi hộp (?) sau khi MC mở hộp rỗng.

 

Bạn đúng ở chỗ nếu chọn đổi, thì xác suất lúc đầu để chọn hộp k là $2/3$. Nhưng xác suất đổi là $1/2$. Nên xác suất nếu chọn đổi và được quà là $1/2*2/3$. Còn nếu bạn không đổi ($1/2$), và bạn được quà, nên lúc đầu phải chọn hộc c ($1/3$), vì vậy xác suất là $1/2*1/3.$ Tổng xác suất để được quà là $1/2.$

 

Tất nhiên, nếu người chơi bắt buộc phải đổi quà, thì bạn đã đúng và đủ.

 

Đọc lại đề thì có câu này

Nhưng sau khi đổi thì xác suất của cái hộp bị mở là bằng 0

 

Như vậy tức là người đã đổi,




#465442 Nghịch lý monty hall

Đã gửi bởi fghost on 20-11-2013 - 10:52 trong Nghịch lý



về phần mình thì mình sẽ chọn đổi để có nhiều cơ hội để trúng hộp có hơn.

Đổi hay không đổi thì cơ hội trúng cũng bằng $1/2$ mà thôi.

 

Chỉ cần bạn nghĩ lại, lúc đó chỉ còn 2 hộp. Bạn không biết hộp nào có quà, việc đổi hay không đổi hoàn toàn giống với việc chọn mới mà thôi.

 

(Nhưng vì sao trong bài của mình, mình lại nói bạn đã giải đúng và đủ, và cho xác suất là $2/3$ sau khi đổi? Là bời vì đó là xác suất trúng với điều kiện người chơi đã đổi. Hiển nhiên không thể so sác xuất có điều kiện đó với xác suất trúng của quyết định đổi hay không đổi được).




#468943 Cho tập $A \subset \mathbb{R}$ bị chặn, chứng m...

Đã gửi bởi fghost on 04-12-2013 - 23:21 trong Giải tích

$-\inf A \geq -x ~ \forall x \in A \rightarrow -\inf A \geq x ~ \forall x \in -A.$ Như vậy $-\inf A$ là chặn trên của $-A$. Với mọi $y < -\inf A,$ $-y> \inf A$, như vậy tồn tại $x' \in A$ sao cho $-y > x'$, hay $y < -x'$, như vậy $y$ không phải chặn trên của $-A$. Như vây $-\inf A= \sup (-A)$

 

Tương tự cho phần kia.




#469208 Cho tập $A \subset \mathbb{R}$ bị chặn, chứng m...

Đã gửi bởi fghost on 06-12-2013 - 08:35 trong Giải tích

bạn có thể trình bày lại đầy đủ đc ko? mình đang làm bài để nộp cho thầy nhưng phần này mình ko hỉu j hết. hic  :wacko:

 

Bạn chỉ cần nhớ $sup$ là chặn trên bé nhất có thể. Để chứng minh số nào đấy là $sup$ (trong trường hợp này là $- \inf A$), thì (1) chứng minh số đấy là chặn trên, và (2) chứng minh mọi số nhỏ hơn số đấy sẽ không phải chặn trên.

 

(1) Chứng minh $-\inf A$ là chặn trên

$-\inf A \geq -x ~ \forall x \in A \rightarrow -\inf A \geq x ~ \forall x \in -A.$ Như vậy $-\inf A$ là chặn trên của $-A$.

(2) Chứng minh mọi số nhỏ hơn $-\inf A$ không phải là chặn trên

Với mọi $y < -\inf A,$ $-y> \inf A$, như vậy tồn tại $x' \in A$ sao cho $-y > x'$ (tính chất của $\inf$, chặn dưới lớn nhất, vì $-y > \inf A$, nên $-y$ không phải là chặn dưới của $A$, như vậy tồn tại $x' < -y$ với $x' \in A$). Hay $y < -x'$, như vậy $y$ không phải chặn trên của $-A$.

 

Như vây $-\inf A= \sup (-A)$

 

Do tập $A \subset \mathbb{R}$ và bị chặn nên ta giả sử $A=(a,b); a,b \in \mathbb{R}$ và $a<b$

 

Từ đó ta được $-A=(-b,-a)$

 

Xét $A$ có $\sup(A)=b;\inf(A)=a$

 

Xét $-A$ có $\sup(-A)=-a;\inf(-A)=-b$

 

Từ đó ta được $\sup(-A)=-\inf(A)=-a;\inf(-A)=-\sup(A)=-b$

 

Cơ sở nào để có thể quy trường hợp tổng quát về trường hợp riêng khi tập hợp chỉ có hai phần tử ? Nếu $A$ hữu hạn thì có thể giả thiết được nhưng $A$ vô hạn thì sao?

 

$A$ bị chặn nên không thể vô hạn.

 

Nhưng vấn đề với việc xem $A=(a,b)$ là vì cấu trúc của $A$ không xác định, và bài toán không phụ thuộc vào cấu trúc của $A$. Nhưng việc coi như $A=(a,b)$ có ích để hình dung cách giải.




#469391 Cho tập $A \subset \mathbb{R}$ bị chặn, chứng m...

Đã gửi bởi fghost on 07-12-2013 - 07:04 trong Giải tích

còn phần inf(-A) = - sup A thì sao ? bạn trình bày luôn ik  :wacko:

 

$\inf(-A)=-\sup(A) \Leftrightarrow -\inf(-A)=\sup(A)$. Theo phần vừa chứng minh, $-\inf(-A)=\sup(-(-A))=\sup(A)$




#469392 5 bài giải tích cổ điển

Đã gửi bởi fghost on 07-12-2013 - 07:21 trong Giải tích

(5) $f$ liên tục, $[a,b]$ compact, connected nên $f([a,b])$ compact, connected. Mà trên $R$ tập compact, connected phải là đoạn $[c,d]$ nào đấy. ĐPCM.

 

(4) $f$ song ánh, khả vi cấp 2, nên $g=f^{-1}$ tồn tại khả vi cấp hai. Ta có $f(g(x))=1.$ Lấy đạo hàm cấp 1, ta có $f'(g(x))g'(x)=1$. Lấy đạo hàm cấp 2, ta có $f''(g(x))g'(x)+f'(g(x))g''(x)=0$.

 

Từ đẳng thức thứ nhất ta có, $g'(x)=\frac{1}{f'(g(x))}$. Từ đẳng thức thứ 2, ta có

$$g''(x)=\frac{-f''(g(x))g'(x)}{f'(g(x))}=\frac{-f''(g(x))}{f'(g(x))}\frac{1}{f'(g(x))}=\frac{-f''(g(x))}{(f'(g(x)))^2}$$




#469596 5 bài giải tích cổ điển

Đã gửi bởi fghost on 08-12-2013 - 04:20 trong Giải tích

bạn ơi, bạn giải lại chi tiết bài 5 giúp mình đc ko? mình doc mà k hỉu, compact, connected là cái gì, mình k có học cái này :(

Tập compact trong $R^n$ tương đương với tập đó đóng và bị chặn.

 

Tập $E$ connected trong $R^n$ là tập mà không thể bị chia ra bởi 2 tập mở $A, B$ trong $R^n$ sao cho $A, B \ne \emptyset,$ $A\cup B=E,$ $A \cap B = \emptyset$.

 

Ánh xạ liên tục sẽ map tập compact đến tập compact, tập connected đến tập connected.




#469815 Tìm số $x$ nhỏ nhất thỏa mãn $f(x)=2001$.

Đã gửi bởi fghost on 09-12-2013 - 00:39 trong Phương trình hàm

Có lẽ đề bài chỉ nên yêu cầu tìm $x \geq 0$ nhỏ nhất.




#470391 5 bài giải tích cổ điển

Đã gửi bởi fghost on 11-12-2013 - 21:15 trong Giải tích

Có lẽ năm nhất chưa học topo.

 

Phần tính chất của hàm liên tục (như map tập compact đến tập compact, tập connected đến tập connected), tùy trường, có lẽ được dạy khi bắt đầu học giải tích ($/varepsilon, \delta$). Không cần dùng topo gì nhiều, nhưng không biết bạn chủ thread có được học hay chưa.




#470428 idean trên vành giao hoán

Đã gửi bởi fghost on 11-12-2013 - 23:55 trong Toán học hiện đại

(1) Gọi $Rad( R )$ Jacobson radical của $R$. $Rad( R )= \bigcap_{\text{m ideal tối đại}} m$. Trong vành $Z/nZ,$ ideal tối đại là $0$ (khi $n=1$), hay $(\bar{p})$ với $p$ ước nguyên tố của $n$, và $\bar{p}$ là ảnh của $p$ trong $\varphi: Z \rightarrow Z/nZ$. Như vậy, với $n=\Pi p_i^{e_i}$ $p_i$ nguyên tố, thì $Rad( R )= (\Pi \bar{p_i})= \Pi \bar{p_i} (Z/nZ)$

 

$Rad( R )=0$ khi $n=1$, hay $(\Pi \bar{p_i}) (Z/nZ)=0 \Leftrightarrow \Pi p_i \subset nZ \Rightarrow n| \Pi p_i$ mà ta có $n=\Pi (p_i)^{e_i}$, vì vậy $n=\Pi p_i$, nói cách khác $n$ squarefree (mọi ước nguyên tố của $n$ chỉ xuất hiện 1 lần).

 

(2)

$$(aZ):_Z (bZ)=\{x \in Z: x(bZ) \subset aZ\}= \{x \in Z: a| xb\}= \frac{lcm(a,b)}{b}Z$$

Vì ta phải tìm $x$ sao cho $a| xb$, mà $a| lcm(a,b)$ và $b| lcm(a,b)$, như vậy $x$ phải là bội của $\frac{lcm(a,b)}{b}$. ($lcm$ là bội chung nhỏ nhất.)

 

(3) Mình không hiểu, bạn muốn trong vành đa thức $Q[X]$, chứng minh $(X^2-3)$ là ideal tối đại?

$Q[X]$ là Euclidean domain, vì có phép chia đa thức, vì vậy là PID (mọi ideal đều được sinh ra bởi 1 phần tử), nên mọi ideal nguyên tố đều là ideal tối đại. Mà $Q[X]$ là UFD (unique factorization domain), nên mọi phần tử irreducible (tối giản) đều nguyên tố. Vì vậy cần chứng minh $X^2-3$ tối giản (irreducible). Mà $X^2-3$ là bậc 2 đa thức, nên nếu có thể đơn giản, thì phải có phần tử ở bậc 1, vì vậy nghiệm phải trong $Q$, nhưng rõ ràng điều này không đúng. Cho nên $X^2-3$ tối giản, $(X^2-3)$ ideal nguyên tố, nên là ideal tối đại.




#470608 Tìm a để (0,0) là điểm liên tục của hàm số sau

Đã gửi bởi fghost on 13-12-2013 - 08:09 trong Giải tích

Đúng như bạn tên-dài-quá-khó-đọc ( :D) đã nói, hàm liên tục chưa chắc đã khả vi. Chỉ cần tìm $\lim f$ khi $(x,y) \rightarrow (0,0)$ là được.

 

Đổi qua polar coordinate, ta được 

$$\lim_{r \rightarrow 0}\frac{2r^2\cos^2(\theta)r\sin(\theta)-r\cos(\theta)r^2\sin^2(\theta)}{r^2}=\lim_{r\rightarrow 0}r(\cos^2(\theta)\sin(\theta)-\cos(\theta)\sin^2(\theta))=0$$




#470822 Tính giới hạn:$\lim_{n \to \infty }nq^{n...

Đã gửi bởi fghost on 14-12-2013 - 01:00 trong Giải tích

$$\sum_{n=1}^{\infty}nq^n=q\sum_{n=1}^{\infty}nq^{n-1}=q\frac{d}{dq}\sum_{n=1}^{\infty}q^n=q\frac{d}{dq}(\frac{1}{1-q}-1)$$

 

Vì tổng hội tụ, nên $\lim_{n\rightarrow \infty} nq^n=0.$




#471393 cmr tồn tại $x_0 \in A,y_0 \in B$ sao cho $d(A,B)=d(...

Đã gửi bởi fghost on 17-12-2013 - 11:55 trong Tôpô

Tham khảo bài này

http://diendantoanho...ng-gian-mêtric/

 

Bổ đề: Với mọi $b \in B$ cho trước, tồn tại $x_0 \in A$ compact, sao cho $d(A,b)=d(x_0,b)$

Chứng minh: tham khảo đường link trên

 

Sau đó, lấy $\inf$ 2 vế trên $b \in B$, ta được, $d(A,B)=d(x_0,B)$

 

Dựa vào bổ đề, ta có với $x_0 \in A$, tồn tại $y_0 \in B$ compact, sao cho $d(B,x_0)=d(x_0,y_0).$

 

Như vậy, $d(A,B)=d(x_0,y_0)$ ĐPCM.